===== Jogo de Menger e $\sigma$-compacidade ===== Provavelmente você vai querer dar uma olhada nos resultados da [[lista:Menger|seção sobre o Jogo de Menger]]. Você provavelmente já viu que se $X$ é um espaço $\sigma$-compacto, então o jogador II tem estratégia vencedora no jogo de Menger. Nesta lista, veremos que se $X$ é um espaço regular com base enumerável, então a recíproca é verdadeira. **~~#~~** Seja $X$ um espaço regular. Mostre que são equivalentes:[[dica:hfechado|Dica]] [[Solucao:hfechado|Solução]] - $X$ é compacto; - Para toda cobertura aberta $\mathcal C$ para $X$, existem $C_1, ..., C_n \in \mathcal C$ tal que $X \subset \overline{\bigcup_{i = 1}^n C_i}$. Para um conjunto $A$ e $n\in\omega$, denotamos por $^{<\omega}A$ coleção das sequências finitas de elementos de $A$, isto é, $$ ^{<\omega}A:=\bigcup_{n\in\omega} {}^n A,$$ onde $^n A$ é a coleção de todas as funções $f:n\rightarrow A$. Se $s=(s_0,\dots,s_n)\in ^{<\omega}A$ e $a\in A$, então $s^{\smallfrown}a:=(s_0,\dots,s_n,a)$. Em particular, note que $\emptyset\in ^{<\omega}A$.\\ **~~#~~** Sejam $X$ um espaço regular, $\mathcal{B}$ uma base de abertos para $X$ e $\mathcal{C}_{\mathcal{B}}$ a coleção de todas as coberturas para $X$ compostas por elementos de $\mathcal{B}$. Se $\sigma$ é uma estratégia para o jogador II no jogo de Menger em $X$, então para $s\in ^{<\omega}\mathcal{C}_{\mathcal{B}}$ qualquer, o conjunto da forma $$K_s:=\bigcap_{\mathcal{U}\in\mathcal{C}_{\mathcal{B}}}\overline{\bigcup\sigma(s^{\smallfrown}\mathcal{U})}$$ é compacto.[[dica:coberturaesperta|Dica]] [[Solucao:KsCompacto|Solução]] **~~#~~** Seguindo as mesmas notações do exercício anterior, suponha agora que $\mathcal{B}$ seja enumerável. **~~#.#~~** Defina $R_{\emptyset}=\{\sigma((\mathcal{U})):\mathcal{U}\in\mathcal{C}_{\mathcal{B}}\}$ e note que $R_\emptyset$ é enumerável. Fixe uma enumeração $\{\sigma((\mathcal{U}_n)):n\in\omega\}$ para $R_\emptyset$, e note que, fazendo $G_0=\bigcap_{n\in\omega}\overline{\bigcup\sigma((\mathcal{U}_n))}$, temos $G_0=K_\emptyset$ (o $K_s$ do exercício anterior para $s=\emptyset$). **~~#.#~~** Para $m\in\omega$ fixado, seja $R_{(m)}:=\{\sigma((\mathcal{U}_m)^{\smallfrown}\mathcal{U}):\mathcal{U}\in\mathcal{C}_{\mathcal{B}}\}$. Novamente, note que $R_{(m)}$ é enumerável, e escolha $\mathcal{U}_{m,n}\in \mathcal{C}_{\mathcal{B}}$ para cada $n\in\omega$ de modo que $R_{(m)}=\{\sigma((\mathcal{U}_m)^\smallfrown\mathcal{U}_{m,n}):n\in\omega\}$. Assim, fazendo $G_{(m)}=\bigcap_{n\in\omega}\overline{\bigcup\sigma((\mathcal{U}_m)^\smallfrown\mathcal{U}_n)}$, temos $G_{(m)}=K_{(\mathcal{U}_m)}$, i.e., $K_s$ para $s=(\mathcal{U}_m)$. **~~#.#~~** Procedendo indutivamente, veja que para cada $s=(s_0,\dots,s_m)\in ^{<\omega}\omega$, existem $\mathcal{U}_{s,n}\in\mathcal{C}_{\mathcal{B}}$ para todo $n\in\omega$ de modo que $G_s=\bigcap_{n\in\omega}\overline{\bigcup{\sigma((\mathcal{U}_{s_0},\dots,\mathcal{U}_{s_m})^{\smallfrown}\mathcal{U}_{s,n})}}.$ **~~#~~** Finalmente, suponha que $\sigma$ seja uma estratégia vencedora para o jogador II no jogo de Menger. Mostre que $X=\bigcup_{s\in ^{<\omega}\omega}G_s$ e conclua que $X$ é $\sigma$-compacto.[[dica:Mengerimplicasigma|Dica]] [[Solucao:MengerImplicaSigma|Solução]] Esta lista foi baseada no artigo [[http://www.ams.org/journals/proc/1995-123-11/S0002-9939-1995-1273523-1/S0002-9939-1995-1273523-1.pdf|A direct proof of a theorem of Telgársky]], de Marion Scheepers.